ayudaaa con estos ejercicios doy CORONA plioss con RESOLUCIÓN urgentee¡

Adjuntos:

Respuestas

Respuesta dada por: Layauwu
0

Respuesta:

Explicación paso a paso:

Respuesta dada por: JainFelix
0

En el ejercicio 16

4( {x}^{3}  + 2) - 3( {x}^{2}  + 1) = P{x}^{3}  + Q{x}^{2}  + R

4 {x}^{3}  + 8 - 3 {x}^{2}   - 3 = P{x}^{3}  + Q{x}^{2}  + R

4{x}^{3} - 3{x}^{2}   + 5= P{x}^{3}  + Q{x}^{2}  + R

Entonces P : 4, Q: -3 y R: 5

Pide hallar

 ({P - Q + R})^{2}

 {(4 - ( - 3) + 5})^{2}

 {12}^{2}  = 144

En el ejercicio 17

Si el polinomio es identicamente nulo, H(x) = 0

Entonces:

El valor de a

(3a - 12) = 0

3a = 12

a = 4

El valor de b

(2b - 8) = 0

2b = 8

b = 4

El valor de c

(c - 7) = 0

c = 7


rosss0: el el ejerciciono hay esa rspst de alternativa
Preguntas similares